III/09/Exercise 9.2

Kirjaudu sisään lähettääksesi tämän lomakkeen

What is the base number of a numeral system if it can only contain the numbers 0, 1, 2 and 3?

Kirjaudu sisään lähettääksesi tämän lomakkeen